1
$a+b+c+abc=4$ koşulunu sağlayan her $a,b,c$ pozitif reelleri için


$$\dfrac{a}{\sqrt{b+c}}+\dfrac{b}{\sqrt{c+a}}+\dfrac{c}{\sqrt{a+b}}\geq \sqrt{2}\left(a+b+c\right)$$


olduğunu gösteriniz.


2
Tüm $a,b,c$ pozitif reelleri için


$$\dfrac{\sqrt{a+b}}{c}+\dfrac{\sqrt{b+c}}{a}+\dfrac{\sqrt{c+a}}{b}\geq \dfrac{4\left(a+b+c\right)}{\sqrt{\left(a+b\right)\left(b+c\right)\left(c+a\right)}}$$


eşitsizliğinin sağlandığını gösteriniz.


3
Tüm $abc=\lambda^3$ koşulunu sağlayan pozitif reel sayılar $a,b,c$ için


$$\dfrac{a^2}{\sqrt{\left(1+a^3\right)\left(1+b^3\right)}}+\dfrac{b^2}{\sqrt{\left(1+b^3\right)\left(1+c^3\right)}}+\dfrac{c^2}{\sqrt{\left(1+c^3\right)\left(1+a^3\right)}}\geq \dfrac{4}{\dfrac{\lambda^6+8}{3\lambda^2\left(3\lambda^2+2\right)}+2}$$


eşitsizliğinin çalıştığını gösteriniz.



4
Genelleştirme 1
$a+b+c+abc=4$ koşulunu sağlayan her $a,b,c$ pozitif reelleri için


$$\dfrac{a}{\sqrt[n]{\lambda_1 b+\lambda_2 c}}+\dfrac{b}{\sqrt[n]{\lambda_1 c+\lambda_2 a}}+\dfrac{c}{\sqrt[n]{\lambda_1 a+\lambda_2 b}}\geq \dfrac{a+b+c}{\sqrt{\lambda_1+\lambda_2}}$$


olduğunu gösteriniz.


5
Genelleştirme 1
Tüm $a_1,a_2,\cdots,a_n$ pozitif reelleri için


$$\sum_{cyc- j}{\dfrac{\sqrt[n-1]{\left(a_ja_{j+1}\cdots a_{j-2}\right)^{n-2}}}{a_{j-1}}}\geq \dfrac{2\left(n-1\right)\sum\limits_{cyc}{a_1}}{\sqrt[n-1]{\prod\limits_{cyc- i}{\left(a_i+a_{i+1}+\cdots+a_{i-2}\right)}}}$$


olduğunu gösteriniz.


6
Genelleştirme 1
Her $x_1,x_2,\cdots,x_n$ reeli ve $k\geq 1$ tam sayısı için


$$\dfrac{x_1}{1+x_1^{2k}}+\dfrac{x_2}{1+x_2^{2k}}+\cdots+\dfrac{x_n}{1+x_n^{2k}}<\sqrt{n^{2k-1}}$$


olduğunu gösteriniz.


7
Genelleştirme 1
Her $x_1,x_2,\cdots,x_{2n+1}>-1$  reeli için


$$\sum_{cyc- j}{\dfrac{1+x_j^2}{n+x_{j+1}+\cdots+x_{j+n}+x_{j+n+1}^2+\cdots+x_{j-1}^2}}\geq \dfrac{2\left(2n+1\right)}{3n}$$


olduğunu gösteriniz.


8
Genelleştirme 1
Her $a_1,a_2,\cdots,a_n$ pozitif reeli için

$$\sum_{cyc- j}{\dfrac{\left(a_j+a_{j+1}+\cdots+a_{j-2}-a_{j-1}\right)^2}{\left(a_j+a_{j+1}+\cdots+a_{j-2}\right)^2+a_{j-1}^2}}\geq \dfrac{23n-54}{25}$$


olduğunu gösteriniz.


9
Genelleştirme 1
Tüm $a_1,a_2,\cdots,a_n$ pozitif reelleri için


$$\sum_{cyc- j}{\left(\dfrac{\left(n-1\right)a_j}{a_{j+1}+a_{j+2}+\cdots+a_{j-1}}\right)^{\dfrac{n-1}{n}}}\geq n$$


olduğunu gösteriniz.


10
Genelleştirme 1
$a^4+b^4+c^4=\lambda^2-1$ koşulunu sağlayan her $a,b,c$ pozitif reelleri için


$$\dfrac{1}{\lambda+2-ab}+\dfrac{1}{\lambda+2-bc}+\dfrac{1}{\lambda+2-ca}\leq \dfrac{1}{2}+\dfrac{4\lambda+1}{2\left(\lambda+1\right)^2}$$


olduğunu gösteriniz.


11
Genelleştirme 1
$\sum\limits_{sym}{a_1a_{2}\cdots a_{n-1}}=n$ koşulunu sağlayan her $a_1,a_2,\cdots,a_n$ pozitif reeli için


$$\sum_{cyc- j}{\dfrac{1}{1+a_j^2\left(\sum\limits_{\substack{1\leq i\leq n\\ i\neq j}}{a_ia_{i+1}\cdots a_{i-3}}\right)}}\leq \dfrac{1}{\prod{a_1}}$$


eşitsizliğinin çalıştığını gösteriniz.


12
Genelleştirme 1
Her $x,y,z$ pozitif reelleri için ve $|r|\geq 1$ tam sayısı için


$$\dfrac{x^3+3\left(r^2-1\right)xy^2}{z^3+x^2y}+\dfrac{y^3+3\left(r^2-1\right)yz^2}{x^3+y^2z}+\dfrac{z^3+3\left(r^2-1\right)zx^2}{y^3+z^2x}\geq \dfrac{3}{2}\left[r+\dfrac{\left(r-1\right)\left(3r+2\right)xyz\left(x+y+z\right)}{2\left(x^3y+y^3z+z^3x\right)}\right]$$


olduğunu gösteriniz.


13
$\prod{a_1}\geq 1$ koşulunu sağlayan her $a_1,a_2,\cdots,a_n$ pozitif reelleri için


$$\sum_{cyc- j}{\dfrac{1}{a_j^n+2a_{j+1}^n+\cdots+\left(n-1\right)a_{j-2}^n+\dfrac{n\left(n+1\right)}{2}}}\leq \dfrac{1}{n}$$


olduğunu gösteriniz.


14
Genelleştirme 1
Her $a_1,a_2,\cdots,a_{2n+1}$ pozitif reelleri için


$$\sum_{cyc- j}{\dfrac{a_j}{\sqrt[k]{\left(a_j+2\left(a_{j+1}+a_{j+2}+\cdots+a_{j+k-1}\right)\right)^{k+1}}}}\geq \dfrac{1}{\sqrt[2n+1]{a_1+a_2+\cdots+a_n}}$$


olduğunu gösteriniz.


15
Genelleştirme 1
Her $a_1,a_2,\cdots,a_n$ reeli için


$$\dfrac{\left(\sum\limits_{cyc}{x_1}\right)^k}{\prod\limits_{cyc- j}{\left(\left(2kx_j\right)^{2k}+4k-1\right)}}\geq \dfrac{1}{\left(4k\right)^{2k}}$$


olduğunu gösteriniz.


16
Genelleştirme 1
$a_1^2+a_2^2+\cdots+a_n^2=n$ eşitliğini sağlayan her $a_1,a_2,\cdots,a_n$ negatif olmayan reelleri için


$$\dfrac{a_1+a_2+\cdots+a_n}{\sqrt{n}}\geq 1+\left(\sqrt{n}-1\right)\sqrt[n]{\left(\prod{a_1}\right)^2}$$


olduğunu gösteriniz.


17
Genelleştirme 1
$a_1,a_2,\cdots,a_n$ pozitif reeller olmak üzere $\sum\limits_{cyc}{a_1}\geq \sum\limits_{cyc}{\dfrac{1}{a_1}}$ ise


$$\sum_{cyc- j}{\dfrac{a_j+a_{j+1}+\cdots+a_{j-2}-a_{j-1}}{a_j^{n}+a_{j+1}^n+\cdots+a_{j-2}^n+\prod{a_1}}}\leq \dfrac{\sum\limits_{sym}{a_1a_2}}{\sum\limits_{cyc- i}{a_ia_{i+1}\cdots a_{i-2}}}$$


olduğunu gösteriniz.


18
Genelleştirme 1
$\prod{a_1}=1$ eşitliğini sağlayan her $a_1,a_2,\cdots,a_n$ pozitif reelleri için


$$\sum_{cyc- j}{\dfrac{1}{a_j+a_{j+1}+\cdots+a_{j-2}+1}}\leq 1$$


olduğunu gösteriniz.


19
Genelleştirme 1
Her  $a_1,a_2,\cdots,a_n$  negatif olmayan reel sayıları için


$$\prod_{cyc- j}{\left(1-a_j+a_j^2\right)}\geq \sqrt{\left(\dfrac{1+\sqrt[n]{\left(\prod{a_1}\right)^4}}{2}\right)^n}$$


olduğunu gösteriniz.


20
Genelleştirme 1
Herhangi $n-1$ tanesi sıfırdan farklı olan  $a_1,a_2,\cdots,a_n$  negatif olmayan reel sayıları için


$$\sum_{cyc- j}{\dfrac{a_j^2}{a_{j+1}^2+a_{j+2}^2+\cdots+a_{j-1}^2}}\geq \sum_{cyc- i}{\dfrac{a_i}{a_{i+1}+a_{i+2}+\cdots+a_{i-1}}}$$


olduğunu gösteriniz.


21
Genelleştirme 1
$p\geq 2$ bir reel sayı olsun. Her $a_1,a_2,\cdots,a_n$ negatif olmayan reelleri için


$$\sum_{cyc- j}{\sqrt[3]{\dfrac{a_j^3+p.\prod{a_1}}{1+p}}} \leq \sqrt[3]{\left(\sum_{cyc}{a_1}\right)\left(\sum_{cyc- i}{a_i^{n-1}+2a_{i+1}a_{i+2}\cdots a_{i-1}}\right)}$$


olduğunu gösteriniz.


22
Her $x_1,x_2,\cdots,x_n$ reeli için


$$\dfrac{x_1}{1+x_1^2}+\dfrac{x_2}{1+x_2^2}+\cdots+\dfrac{x_n}{1+x_n^2}<\sqrt{n}$$


olduğunu gösteriniz.


23
Her $a,b,c$ pozitif reeli için


$$\dfrac{\left(a+b-c\right)^2}{\left(a+b\right)^2+c^2}+\dfrac{\left(b+c-a\right)^2}{\left(b+c\right)^2+a^2}+\dfrac{\left(c+a-b\right)^2}{\left(c+a\right)^2+b^2}\geq \dfrac{3}{5}$$


olduğunu gösteriniz.


24
Tüm $a,b,c$ pozitif reelleri için


$$\left(\dfrac{2a}{b+c}\right)^{\dfrac{2}{3}}+\left(\dfrac{2b}{c+a}\right)^{\dfrac{2}{3}}+\left(\dfrac{2c}{a+b}\right)^{\dfrac{2}{3}}\geq 3$$


olduğunu gösteriniz.


25
$a^4+b^4+c^4=3$ koşulunu sağlayan her $a,b,c$ pozitif reelleri için


$$\dfrac{1}{4-ab}+\dfrac{1}{4-bc}+\dfrac{1}{4-ca}\leq 1$$


olduğunu gösteriniz.


26
$ab+bc+ca=3$ koşulunu sağlayan her $a,b,c$ pozitif reeli için


$$\dfrac{1}{1+a^2\left(b+c\right)}+\dfrac{1}{1+b^2\left(c+a\right)}+\dfrac{1}{1+c^2\left(a+b\right)}\leq \dfrac{1}{abc}$$


eşitsizliğinin çalıştığını gösteriniz.


27
Her $x,y,z$ pozitif reelleri için


$$\dfrac{x^3}{z^3+x^2y}+\dfrac{y^3}{x^3+y^2z}+\dfrac{z^3}{y^3+z^2x}\geq \dfrac{3}{2}$$


olduğunu gösteriniz.


28
$abc\geq 1$ koşulunu sağlayan her $a,b,c$ pozitif reelleri için


$$\dfrac{1}{a^3+2b^3+6}+\dfrac{1}{b^3+2c^3+6}+\dfrac{1}{c^3+2a^3+6}\leq \dfrac{1}{3}$$


olduğunu gösteriniz.


29
Her $a,b,c$ pozitif reelleri için


$$\dfrac{a}{\sqrt{\left(a+2b\right)^3}}+\dfrac{b}{\sqrt{\left(b+2c\right)^3}}+\dfrac{c}{\sqrt{\left(c+2a\right)^3}}\geq \dfrac{1}{\sqrt{a+b+c}}$$


olduğunu gösteriniz.


30
Her $a,b$ reeli için


$$\dfrac{a+b}{\left(4a^2+3\right)\left(4b^2+3\right)}$$


ifadesinin maksimum değerini bulunuz.


31
$a^2+b^2+c^2+d^2=4$ eşitliğini sağlayan her $a,b,c,d$ negatif olmayan reel sayıları için


$$\dfrac{a+b+c+d}{2}\geq 1+\sqrt{abcd}$$


olduğunu gösteriniz.


32
$abc=1$ koşulunu sağlayan her $a,b,c$ pozitif reeli için


$$\dfrac{1}{a+b+1}+\dfrac{1}{b+c+1}+\dfrac{1}{c+a+1}\leq 1$$


eşitsizliğinin çalıştığını gösteriniz.


33
Her  $a,b,c,d$  negatif olmayan reel sayıları için


$$\left(1-a+a^2\right)\left(1-b+b^2\right)\left(1-c+c^2\right)\left(1-d+d^2\right)\geq \left(\dfrac{1+abcd}{2}\right)^2$$


olduğunu gösteriniz.


34
$p\geq 2$ bir reel sayı olsun. Her $a,b,c$ negatif olmayan reelleri için


$$\sqrt[3]{\dfrac{a^3+pabc}{1+p}}+\sqrt[3]{\dfrac{b^3+pabc}{1+p}}+\sqrt[3]{\dfrac{c^3+pabc}{1+p}}\leq a+b+c$$


olduğunu gösteriniz.


35
Herhangi ikisi sıfırdan farklı  $a,b,c$  negatif olmayan reel sayıları için


$$\dfrac{a^2}{b^2+c^2}+\dfrac{b^2}{c^2+a^2}+\dfrac{c^2}{a^2+b^2}\geq \dfrac{a}{b+c}+\dfrac{b}{c+a}+\dfrac{c}{a+b}$$


olduğunu gösteriniz.


36
$ab+bc+ca=3$ eşitliğini sağlayan her $a,b,c$ negatif olmayan reeller için


$$\dfrac{1}{a^2+1}+\dfrac{1}{b^2+1}+\dfrac{1}{c^2+1}\geq \dfrac{3}{2}$$


olduğunu gösteriniz.



Sitemap 1 2 3 4 5 6 7 8 9 10 11 12 13 14 15 16 17 18 19 20 21 22 23 24 25 26 27 28 29 30 31 32 33 34 35 36 37 
SimplePortal 2.3.3 © 2008-2010, SimplePortal